Maven第3篇:详解maven解决依赖问题

本文主要内容

  1. 感受一下maven的效果
  2. maven约定配置
  3. maven中pom文件
  4. maven坐标详解
  5. maven依赖导入功能
  6. maven依赖范围详解
  7. maven依赖的传递
  8. maven中依赖调解功能
  9. 可选依赖(optional元素)的使用
  10. 排除依赖的使用

先来感受一下maven的神奇

安装maven3.6.10

idea中配置maven

使用mven创建一个springboot项目

运行maven项目

项目路径下,执行以下命令
mvn spring-boot:run

约定配置

Maven 提倡使用一个共同的标准目录结构,Maven 使用约定优于配置的原则,大家尽可能的遵守这样的目录结构,如下所示:

[外链图片转存失败,源站可能有防盗链机制,建议将图片保存下来直接上传(img-GemWEMVP-1661155142806)(data:image/png;base64,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)]

结合上面表格中的信息,再去看看springboot-chat01项目的结构,这是maven项目标准的结构,大家都按照这个约定来,然后maven中打包、运行、部署时候就非常方便了,maven他自己就知道你项目的源码、资源、测试代码、打包输出的位置,这些都是maven规定好的,就不是你随意搞的一个结构,所以不需要我们再去配置了,所以使用maven去打包、部署、运行都是非常方便的。

这块现实中也有很多案例,比如USB接口,电压,这些都是规定好的,如果USB接口所有厂商制造的大小都不一致,那我们使用电子设备的时候是相当难受的。

pom文件

当我们在项目中需要用到maven帮我们解决jar包依赖问题,帮我们解决项目中的编译、测试、打包、部署时,项目中必须要有pom.xml文件,这些都是依靠pom的配置来完成的。

POM( Project Object Model,项目对象模型 ) 是 Maven 工程的基本工作单元,是一个XML文件,包含了项目的基本信息,用于描述项目如何构件,声明项目依赖,等等。

执行任务或目标时,Maven 会在当前目录中查找 POM。它读取 POM,获取所需的配置信息,然后执行目标。

POM 中可以指定以下配置:

  • 项目依赖
  • 插件
  • 执行目标
  • 项目构件 profile
  • 项目版本
  • 项目开发者列表
  • 相关邮件列表信息

在创建 POM 之前,我们首先需要描述项目组 (groupId),项目的唯一ID。

<?xml version="1.0" encoding="UTF-8"?>
<project xmlns="http://maven.apache.org/POM/4.0.0"xmlns:xsi="http://www.w3.org/2001/XMLSchema-instance"xsi:schemaLocation="http://maven.apache.org/POM/4.0.0 http://maven.apache.org/xsd/maven-4.0.0.xsd"><!-- 模型版本 --><modelVersion>4.0.0</modelVersion><!-- 定义当前构件所属的组,通常与域名反向一一对应 --><groupId>com.javacode2018</groupId><!--项目的唯一ID,一个groupId下面可能多个项目,就是靠artifactId来区分的--><artifactId>maven-chat02</artifactId><!-- 版本号 --><version>1.0-SNAPSHOT</version></project>

maven坐标

maven中引入了坐标的概念,每个构件都有唯一的坐标,我们使用maven创建一个项目需要标注其坐标信息,而项目中用到其他的一些构件,也需要知道这些构件的坐标信息。

maven中构件坐标是通过一些元素定义的,他们是groupId、artifactId、version、packaging、classifier,如我们刚刚上面创建的springboot项目,它的pom中坐标信息如下:

<groupId>com.javacode2018</groupId>
<artifactId>springboot-chat01</artifactId>
<version>0.0.1-SNAPSHOT</version>
<packaging>jar</packaging>

goupId:定义当前构件所属的组,通常与域名反向一一对应。

artifactId:项目组中构件的编号。

version:当前构件的版本号,每个构件可能会发布多个版本,通过版本号来区分不同版本的构件。

package:定义该构件的打包方式,比如我们需要把项目打成jar包,采用java -jar去运行这个jar包,那这个值为jar;若当前是一个web项目,需要打成war包部署到tomcat中,那这个值就是war,可选(jar、war、ear、pom、maven-plugin),比较常用的是jar、war、pom,这些后面会详解。

上面接元素中,groupId、artifactId、version是必须要定义的,packeage可以省略,默认为jar。

我们可以将上面创建的springboot项目发布出去,然后只需要告诉别人springboot-chat01这个项目的坐标信息,其他人就可以直接使用了,而且其他人用的时候,不用关心springboot-chat01运行需要依赖什么,springboot-chat01依赖的这些都会自动导入,非常方便。

maven导入依赖的构件

maven可以帮我们引入需要依赖的构件(jar等),而maven是如何定位到某个构件的呢?

项目中如果需要使用第三方的jar,我们需要知道其坐标信息,然后将这些信息放入pom.xml文件中的dependencies元素中:

<project><dependencies><!-- 在这里添加你的依赖 --><dependency><groupId></groupId><artifactId></artifactId><version></version><type></type><scope></scope><optional></optional><exclusions><exclusion></exclusion><exclusion></exclusion></exclusions></dependency></dependencies>
</project>
  • dependencies元素中可以包含多个dependency,每个dependency就表示当前项目需要依赖的一个构件的信息
  • dependency中groupId、artifactId、version是定位一个构件必须要提供的信息,所以这几个是必须的
  • type:依赖的类型,表示所要依赖的构件的类型,对应于被依赖的构件的packaging。大部分情况下,该元素不被声明,默认值为jar,表示被依赖的构件是一个jar包。
  • scope:依赖的范围,后面详解
  • option:标记依赖是否可选,后面详解
  • exclusions:用来排除传递性的依赖

通常情况下我们依赖的都是一些jar包,所以大多数情况下,只需要提供groupId、artifactId、version信息就可以了。

创建一个maven项目(maven-chat02)

打开idea,点击File->New->Project,选择Maven

点击Next,输入项目的maven坐标信息

点击Next->Finish,完成项目的创建

打开pom.xml,引入springmvc依赖

<dependency><groupId>org.springframework</groupId><artifactId>spring-web</artifactId><version>5.2.1.RELEASE</version>
</dependency>

pom.xml文件最终如下:

<?xml version="1.0" encoding="UTF-8"?>
<project xmlns="http://maven.apache.org/POM/4.0.0"xmlns:xsi="http://www.w3.org/2001/XMLSchema-instance"xsi:schemaLocation="http://maven.apache.org/POM/4.0.0 http://maven.apache.org/xsd/maven-4.0.0.xsd"><modelVersion>4.0.0</modelVersion><groupId>com.javacode2018</groupId><artifactId>maven-chat02</artifactId><version>1.0-SNAPSHOT</version><dependencies><dependency><groupId>org.springframework</groupId><artifactId>spring-web</artifactId><version>5.2.1.RELEASE</version></dependency></dependencies></project>

可以看到我们只引入了一个spring-web的jar包,springmvc还需要spring-core等jar包的支持,我们来看一下,maven是否帮我们也导入了。

pom.xml文件中,右键选中Show Dependencies,可以显示当前项目依赖的jar包

图中显示了项目中的依赖信息,箭头有多级,从左向右,第一个箭头表示第一级,是直接依赖,后面的都是间接依赖,属于传递性依赖。

我们在maven-chat02中添加了spring-web的依赖,并没有引入spring-beans、spring-core、spring-jcl的依赖,但是maven都自动帮我们导入了,这是因为spring-web的pom.xml中定义了它自己的依赖,当我们使用spring-web的时候,spring-web需要依赖的jar也会自动被依赖进来,maven是不是很强大。

如果没有maven,我们找jar是相当痛苦的,经常会出现少添加了一些jar,或者依赖的jar版本对不上等问题,而maven直接帮我们解决了。

maven依赖范围(scope)

我们的需求:

  1. 我们在开发项目的过程中,可能需要用junit来写一些测试用例,此时需要引入junit的jar包,但是当我们项目部署在线上运行了,测试代码不会再执行了,此时junit.jar是不需要了,所以junit.jar只是在编译测试代码,运行测试用例的时候用到,而上线之后用不到了,所以部署环境中是不需要的
  2. 我们开发了一个web项目,在项目中用到了servlet相关的jar包,但是部署的时候,我们将其部署在tomcat中,而tomcat中自带了servlet的jar包,那么我们的需求是开发、编译、单元测试的过程中需要这些jar,上线之后,servlet相关的jar由web容器提供,也就是说打包的时候,不需要将servlet相关的jar打入war包了
  3. 像jdbc的驱动,只有在运行的时候才需要,编译的时候是不需要的

这些需求怎么实现呢?

我们都知道,java中编译代码、运行代码都需要用到classpath变量,classpath用来列出当前项目需要依赖的jar包,这块不清楚的可以去看一下classpath和jar。

maven用到classpath的地方有:编译源码、编译测试代码、运行测试代码、运行项目,这几个步骤都需要用到classpath。

如上面的需求,编译、测试、运行需要的classpath对应的值可能是不一样的,这个maven中的scope为我们提供了支持,可以帮我们解决这方面的问题,scope是用来控制被依赖的构件与classpath的关系(编译、打包、运行所用到的classpath),scope有以下几种值:

compile

编译依赖范围,如果没有指定,默认使用该依赖范围,对于编译源码、编译测试代码、测试、运行4种classpath都有效,比如上面的spring-web。

test

测试依赖范围,使用此依赖范围的maven依赖,只对编译测试、运行测试的classpath有效,在编译主代码、运行项目时无法使用此类依赖。比如junit,它只有在编译测试代码及运行测试的时候才需要。

provide

已提供依赖范围。表示项目的运行环境中已经提供了所需要的构件,对于此依赖范围的maven依赖,对于编译源码、编译测试、运行测试中classpath有效,但在运行时无效。比如上面说到的servlet-api,这个在编译和测试的时候需要用到,但是在运行的时候,web容器已经提供了,就不需要maven帮忙引入了。

runtime

运行时依赖范围,使用此依赖范围的maven依赖,对于编译测试、运行测试和运行项目的classpath有效,但在编译主代码时无效,比如jdbc驱动实现,运行的时候才需要具体的jdbc驱动实现。

system

系统依赖范围,该依赖与3中classpath的关系,和provided依赖范围完全一致。但是,使用system范围的依赖时必须通过systemPath元素显示第指定依赖文件的路径。这种依赖直接依赖于本地路径中的构件,可能每个开发者机器中构件的路径不一致,所以如果使用这种写法,你的机器中可能没有问题,别人的机器中就会有问题,所以建议谨慎使用。

如下:

<dependency><groupId>com.javacode2018</groupId><artifactId>rt</artifactId><version>1.8</version><scope>system</scope><systemPath>${java.home}/lib/rt.jar</systemPath>
</dependency>

import

这个比较特殊,后面的文章中单独讲,springboot和springcloud中用到的比较多。

依赖范围与classpath的关系如下:

[外链图片转存失败,源站可能有防盗链机制,建议将图片保存下来直接上传(img-iQ3rAjcg-1661155142808)(data:image/png;base64,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)]

scope如果对于运行范围有效,意思是指依赖的jar包会被打包到项目的运行包中,最后运行的时候会被添加到classpath中运行。如果scope对于运行项目无效,那么项目打包的时候,这些依赖不会被打包到运行包中。

依赖的传递

上面我们创建的maven-chat02中依赖了spring-web,而我们只引入了spring-web依赖,而spring-web又依赖了spring-beans、spring-core、spring-jcl,这3个依赖也被自动加进来了,这种叫做依赖的传递。

不过上面我们说的scope元素的值会对这种传递依赖会有影响。

假设A依赖于B,B依赖于C,我们说A对于B是第一直接依赖,B对于C是第二直接依赖,而A对于C是传递性依赖,而第一直接依赖的scope和第二直接依赖的scope决定了传递依赖的范围,即决定了A对于C的scope的值。

下面我们用表格来列一下这种依赖的效果,表格最左边一列表示第一直接依赖(即A->B的scope的值),而表格中的第一行表示第二直接依赖(即B->C的scope的值),行列交叉的值显示的是A对于C最后产生的依赖效果。

[外链图片转存失败,源站可能有防盗链机制,建议将图片保存下来直接上传(img-uiEQ5fKS-1661155142809)(data:image/png;base64,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)]

解释一下:

  1. 比如A->B的scope是compile,而B->C的scope是test,那么按照上面表格中,对应第2行第3列的值-,那么A对于C是没有依赖的,A对C的依赖没有从B->C传递过来,所以A中是无法使用C的
  2. 比如A->B的scope是compile,而B->C的scope是runtime,那么按照上面表格中,对应第2行第5列的值为runtime,那么A对于C是的依赖范围是runtime,表示A只有在运行的时候C才会被添加到A的classpath中,即对A进行运行打包的时候,C会被打包到A的包中
  3. 大家仔细看一下,上面的表格是有规律的,当B->C依赖是compile的时候(表中第2列),那么A->C的依赖范围和A->B的sope是一样的;当B->C的依赖是test时(表中第3列),那么B->C的依赖无法传递给A;当B->C的依赖是provided(表第4列),只传递A->C的scope为provided的情况,其他情况B->C的依赖无法传递给A;当B->C的依赖是runtime(表第5列),那么C按照B->C的scope传递给A

maven依赖调解功能

现实中可能存在这样的情况,A->B->C->Y(1.0),A->D->Y(2.0),此时Y出现了2个版本,1.0和2.0,此时maven会选择Y的哪个版本?

解决这种问题,maven有2个原则:

路径最近原则

上面A->B->C->Y(1.0),A->D->Y(2.0),Y的2.0版本距离A更近一些,所以maven会选择2.0。

但是如果出现了路径是一样的,如:A->B->Y(1.0),A->D->Y(2.0),此时maven又如何选择呢?

最先声明原则

如果出现了路径一样的,此时会看A的pom.xml中所依赖的B、D在dependencies中的位置,谁的声明在最前面,就以谁的为主,比如A->B在前面,那么最后Y会选择1.0版本。

可选依赖(optional元素)

有这么一种情况:

A->B中scope:compile
B->C中scope:compile

按照上面介绍的依赖传递性,C会传递给A,被A依赖。

假如B不想让C被A自动依赖,可以怎么做呢?

dependency元素下面有个optional,是一个boolean值,表示是一个可选依赖,B->C时将这个值置为true,那么C不会被A自动引入。

排除依赖

A项目的pom.xml中

<dependency><groupId>com.javacode2018</groupId><artifactId>B</artifactId><version>1.0</version>
</dependency>

B项目1.0版本的pom.xml中

<dependency><groupId>com.javacode2018</groupId><artifactId>C</artifactId><version>1.0</version>
</dependency>

上面A->B的1.0版本,B->C的1.0版本,而scope都是默认的compile,根据前面讲的依赖传递性,C会传递给A,会被A自动依赖,但是C此时有个更新的版本2.0,A想使用2.0的版本,此时A的pom.xml中可以这么写:

<dependency><groupId>com.javacode2018</groupId><artifactId>B</artifactId><version>1.0</version><exclusions><exclusion><groupId>com.javacode2018</groupId><artifactId>C</artifactId></exclusion></exclusions>
</dependency>

上面使用使用exclusions元素排除了B->C依赖的传递,也就是B->C不会被传递到A中。

factId>B
1.0

com.javacode2018
C

上面使用使用exclusions元素排除了B->C依赖的传递,也就是B->C不会被传递到A中。

exclusions中可以有多个exclusion元素,可以排除一个或者多个依赖的传递,声明exclusion时只需要写上groupId、artifactId就可以了,version可以省略。

Maven第3篇:详解maven解决依赖问题相关推荐

  1. idea 执行java maven,IDEA的run maven方式启动步骤详解

    安装jetty插件 1. 找到Plugins,查找jetty插件,安装"IDEA Jetty Runner",安装好后重启IDEA 安装插件:Maven Helper 方法同Jet ...

  2. 你真的了解Maven pom.xml 的配置吗?【详解maven pom】

    Maven POM POM( Project Object Model,项目对象模型 ) 是 Maven 工程的基本工作单元,是一个XML文件,包含了项目的基本信息,用于描述项目如何构建,声明项目依赖 ...

  3. Maven中pom文件详解

    在IDE中使用Maven IDE工具 MyEclipse 集成maven插件 Eclipse ​ Eclipse For Java EE IDEA 集成Maven插件 在IDE中 配置Maven 指定 ...

  4. Maven中央仓库发布详解

    Maven中央仓库发布详解 开通官方账号 注册 提交issue 按照提示步骤操作 准备工作 签名工具 配置Maven Setting.xml 上传jar包 修改项目pom.xml 仓库管理 开通官方账 ...

  5. html 5效果不显示,详解如何解决H5开发使用wx.hideMenuItems无效果不生效

    情况:引入SDK 的签名不报错与调试工具生成的结果也是一模一样,但是使用hideMenuItems没有小效果,不会报错. 解决方式:把要执行的wx.hideMenuItems()放到wx.ready这 ...

  6. python gil 解除_详解Python中的GIL(全局解释器锁)详解及解决GIL的几种方案

    先看一道GIL面试题: 描述Python GIL的概念, 以及它对python多线程的影响?编写一个多线程抓取网页的程序,并阐明多线程抓取程序是否可比单线程性能有提升,并解释原因. GIL:又叫全局解 ...

  7. postionfixed固定_详解如何解决position:fixed固定定位偏移问题

    问题 css固定定位position:fixed很容易使用,就是相对浏览器的viewport进行定位,top:0;left:0就是在左上角. .container{ width: 100px; hei ...

  8. css鼠标拖拉卡顿_详解overflow-scrolling解决滚动卡顿问题

    前言 如果你对某个div或模块使用了overflow: scroll属性,在iOS系统的手机上浏览时,则会出现明显的卡顿现象.但是在android系统的手机上则不会出现该问题. 解决方法 以下代码可解 ...

  9. PHP开发中常见的安全问题详解和解决方法

    PHP开发中常见的安全问题详解和解决方法 参考文章: (1)PHP开发中常见的安全问题详解和解决方法 (2)https://www.cnblogs.com/walblog/articles/83313 ...

最新文章

  1. ASCII计算机语言,unicode和ascii的区别是什么
  2. 磁铁驱动反向续流串接电阻的的分析
  3. java代码连接oracle数据库连接_在JAVA中连接Oracle数据库(例子)
  4. x-lite asterisk 成功实现视频通话
  5. Mysql中Innodb大量插入数据时SQL语句的优化
  6. 漫步最优化二十七——二次插值法
  7. android应用开发(23)---处理Activity状态更改
  8. Java中在时间戳计算的过程中遇到的数据溢出问题
  9. java框架ssm面试题2016_Java面试-框架篇(SSM-SpringMVC)
  10. 大事件归来,爷青回!
  11. 重启docker容器命令
  12. C++的操作符delete很特殊,跟new不对称
  13. MySQL备份与恢复-innobackupex
  14. 第十五章、实现属性以访问字段
  15. TextCNN pytorch实现
  16. 计网 - TCP 的稳定性:滑动窗口和流速控制是怎么回事?
  17. python爬取b站视频
  18. HOOK SSDT 实现内核级的进程保护
  19. erlang send_after 源码剖析
  20. 真过份,女友居然要我坐在马桶上小便

热门文章

  1. 解决Antimalware Service Executable(windows defender)占用过高CPU和内存的方法
  2. php创建多级栏目_PHP 实现无限极栏目分类
  3. HyperLPR车牌识别库代码分析(9)
  4. vim复制粘贴的命令
  5. libGDX的启动类和配置
  6. Layui数据表格隔行变色的两种方法
  7. 【LiteApp系列】爱奇艺小程序架构浅析
  8. Linux下的FTP安装和登录
  9. python中scapy模块的使用
  10. mac下实用的代码编辑器